Oblicz granicę

Wyznaczanie granic funkcji. Ciągłość w punkcie i ciągłość jednostajna na przedziale. Reguła de l'Hospitala.
max123321
Użytkownik
Użytkownik
Posty: 3394
Rejestracja: 26 maja 2016, o 01:25
Płeć: Mężczyzna
Lokalizacja: Kraków
Podziękował: 981 razy
Pomógł: 3 razy

Oblicz granicę

Post autor: max123321 »

Oblicz granicę:
\(\displaystyle{ \lim_{n \to \infty} \int_{0}^{\infty} \frac{1}{1+x^n} }\)

Proszę o sprawdzenie poniższego rozwiązania:
Chcę skorzystać z twierdzenia o zbieżności ograniczonej (zmajoryzowanej) i znajduję funkcję \(\displaystyle{ g:\left[0,\infty \right) \rightarrow \RR}\) daną wzorem \(\displaystyle{ g(x)=2}\). Ciąg funkcyjny \(\displaystyle{ f_n(x)= \frac{1}{1+x^n} \le 2}\) dla każdego \(\displaystyle{ x \in \left[0,\infty\right) }\). Zatem można korzystać z twierdzenia o zbieżności ograniczonej. Zauważmy, że \(\displaystyle{ f(x)= \lim_{n \to \infty} \frac{1}{1+x^n}= \begin{cases} 1 \text{ dla } x \in \left[ 0,1\right)\\1/2 \text{ dla } x=1 \\ 0 \text{ dla } x>1 \end{cases} }\), zatem:
\(\displaystyle{ \lim_{n \to \infty} \int_{0}^{\infty} \frac{1}{1+x^n}= \int_{0}^{\infty}f(x)= \int_{0}^{1}1dx+ \int_{1}^{\infty}0dx=1 }\)

Czy tak jest dobrze?
Tmkk
Użytkownik
Użytkownik
Posty: 1718
Rejestracja: 15 wrz 2010, o 15:36
Płeć: Mężczyzna
Lokalizacja: Ostrołęka
Podziękował: 59 razy
Pomógł: 501 razy

Re: Oblicz granicę

Post autor: Tmkk »

Niestety, funkcja stale równa \(\displaystyle{ 2}\) nie jest dobrą majorantą, bo nie jest całkowalna.
Awatar użytkownika
Premislav
Użytkownik
Użytkownik
Posty: 15687
Rejestracja: 17 sie 2012, o 13:12
Płeć: Mężczyzna
Lokalizacja: Warszawa
Podziękował: 196 razy
Pomógł: 5221 razy

Re: Oblicz granicę

Post autor: Premislav »

No zaraz, ale przecież każdą funkcję można ograniczyć z góry przez jakąś inną funkcję, co miałoby wynikać z ograniczenia \(\displaystyle{ \frac{1}{1+x^{n}}\le 2}\) na \(\displaystyle{ [0,\infty]}\), skoro \(\displaystyle{ g(x)=2}\) nie jest całkowalna na \(\displaystyle{ [0, \infty]}\).
Pomysł, żeby skorzystać z twierdzenia Lebesgue'a o zbieżności ograniczonej jest dobry (choć można sobie poradzić i bez tego), ale wykonanie nie. Wskazówka: jeśli oznaczymy \(\displaystyle{ g_{n}(x)=\frac{1}{1+x^{n}}}\), to na odcinku \(\displaystyle{ [1,+\infty]}\) mamy nierówność \(\displaystyle{ |g_{n}(x)|\le g_{2}(x)}\) dla każdego \(\displaystyle{ n\ge 2}\), a \(\displaystyle{ g_{2}}\) łatwo scałkować. No i pozostaje jakkolwiek zmajoryzować (nawet przez stałą) na odcinku \(\displaystyle{ [0,1]}\).

Dodano po 6 minutach 21 sekundach:
Ej no, kachna mać, zdublowałeś wątki. Tak to się nie bawię, dodaję do ignorowanych, pozdro.
Ostatnio zmieniony 11 wrz 2020, o 08:18 przez Dasio11, łącznie zmieniany 1 raz.
Powód: Przeniesiono ze zdublowanego wątku.
Awatar użytkownika
Janusz Tracz
Użytkownik
Użytkownik
Posty: 4069
Rejestracja: 13 sie 2016, o 15:01
Płeć: Mężczyzna
Lokalizacja: hrubielowo
Podziękował: 80 razy
Pomógł: 1393 razy

Re: Oblicz granicę

Post autor: Janusz Tracz »

Łatwo widać, że dla \(\displaystyle{ n \ge 3}\):

\(\displaystyle{ \frac{1}{2} \cdot \left( \frac{1}{1+\left( \sqrt[n]{ \frac{n-1}{n+1}} - \frac{1}{ \sqrt{n} }\right)^n} -1\right) \cdot \left( \sqrt[n]{ \frac{n-1}{n+1}} - \frac{1}{ \sqrt{n} }\right) +\left(\sqrt[n]{ \frac{n-1}{n+1}} - \frac{1}{ \sqrt{n} } \right) \le \blue{\int_{0}^{ \infty } \frac{1}{1+x^n} \dd x} \le 1+ \frac{1}{n-1} }\)

Twierdzenie o trzech ciągach załatwia sprawę. Dowód pozostawiam czytelnikowi.

A tak na serio to:

Niech \(\displaystyle{ n \ge 3}\) oraz \(\displaystyle{ f_n(x)= \frac{1}{1+x^n} }\) wtedy:

\(\displaystyle{ \int_{0}^{1} \frac{1}{1+x^n} \dd x \le \int_{0}^{ \infty } \frac{1}{1+x^n} \dd x \le \int_{0}^{1} 1 \dd x +\int_{1}^{ \infty } \frac{1}{x^n} \dd x}\)
czyli:
\(\displaystyle{ \int_{0}^{1} \frac{1}{1+x^n} \dd x \le \int_{0}^{ \infty } \frac{1}{1+x^n} \dd x \le 1+ \frac{1}{n-1} }\)

Ale ponieważ \(\displaystyle{ \frac{1}{1+x^n}}\) jest wklęsłą na \(\displaystyle{ \left[ 0, \sqrt[n]{ \frac{n-1}{n+1}} \right] \subset \left[ 0,1\right] }\) i dodatnia na \(\displaystyle{ \left[ 0,1\right] }\) to jest też wklęsła na \(\displaystyle{ \left[ 0, \sqrt[n]{ \frac{n-1}{n+1}} - \frac{1}{ \sqrt{n} } \right] \subset \left[ 0, \sqrt[n]{ \frac{n-1}{n+1}} \right] }\) zatem na \(\displaystyle{ \left[ 0, \sqrt[n]{ \frac{n-1}{n+1}} - \frac{1}{ \sqrt{n} } \right] }\) zachodzi:


\(\displaystyle{ \int_{0}^{\sqrt[n]{ \frac{n-1}{n+1}} - \frac{1}{ \sqrt{n} }} \text{prosta przechodząca przez } \left( 0,f(0)\right) \text{ oraz }\left( \sqrt[n]{ \frac{n-1}{n+1}} - \frac{1}{ \sqrt{n} },f_n\left( \sqrt[n]{ \frac{n-1}{n+1}} - \frac{1}{ \sqrt{n} }\right) \right) \dd x \le \int_{0}^{1} \frac{1}{1+x^n} \dd x }\)

Oznaczywszy \(\displaystyle{ N=\sqrt[n]{ \frac{n-1}{n+1}} - \frac{1}{ \sqrt{n} }}\) można zapisać:


\(\displaystyle{ \int_{0}^{N} \left( \frac{f(N)-1}{N} x+1\right) \dd x \le \int_{0}^{1} \frac{1}{1+x^n} \dd x }\)


\(\displaystyle{ \frac{f_n(N)-1}{2} \cdot N+N \le \int_{0}^{1} \frac{1}{1+x^n} \dd x }\)

Czyli ostatecznie mamy:

\(\displaystyle{ \frac{f_n(N)-1}{2} \cdot N+N \le \int_{0}^{ \infty } \frac{1}{1+x^n} \dd x \le 1+ \frac{1}{n-1} }\)

Odnotujmy teraz dwie rzeczy:

\(\displaystyle{ \bullet}\) Dla \(\displaystyle{ n \rightarrow \infty }\) mamy \(\displaystyle{ N\to 1}\)

\(\displaystyle{ \bullet}\) Dla \(\displaystyle{ n \rightarrow \infty }\) mamy \(\displaystyle{ f_n(N)\to 1}\)

Pierwsze spostrzeżenie nie wymaga komentarza, drugiego natomiast dowodzimy:
\(\displaystyle{ \lim_{n \to \infty }f_n(N)=\lim_{n \to \infty } \frac{1}{1+\left(\sqrt[n]{ \frac{n-1}{n+1}} - \frac{1}{ \sqrt{n} } \right)^n } = 1}\)
wszak
\(\displaystyle{ 0\le \left(\sqrt[n]{ \frac{n-1}{n+1}} - \frac{1}{ \sqrt{n} } \right)^n \le \left(1 - \frac{1}{ \sqrt{n} } \right)^n \rightarrow 0 }\)

Zauważamy teraz dlaczego zdefiniowałem \(\displaystyle{ N}\) jako \(\displaystyle{ \sqrt[n]{ \frac{n-1}{n+1}} - \red{\frac{1}{ \sqrt{n} }}}\) a nie rozważałem sieczną na całym zbiorze wklęsłości. Widać, że ta część \(\displaystyle{ \red{\frac{1}{ \sqrt{n} }}}\) okazała się niezbędna do skutecznego szacowania. Wracamy teraz do nierówności:

\(\displaystyle{ \frac{f_n(N)-1}{2} \cdot N+N \le \int_{0}^{ \infty } \frac{1}{1+x^n} \dd x \le 1+ \frac{1}{n-1} }\)

która przy \(\displaystyle{ n \rightarrow \infty }\) i powołaniu się na tw. o trzech ciągach daje wniosek, że:

\(\displaystyle{ \lim_{ n\to \infty } \int_{0}^{ \infty } \frac{1}{1+x^n} \dd x=1}\)
ODPOWIEDZ